PT26.S3.Q24 - Could a kind soul answer my OG question?

Ashley2018-1Ashley2018-1 Alum Member
edited July 2021 in Logical Reasoning 2249 karma

I think I get it? Non-individuals can buy cars too but what if answer choice e had stated that the proportion of individuals and non-individuals (i.e., corporations etc) purchasing cars were about the same (50/50) Would that make answer choice e incorrect or just less strong of a weakener? And why? Couldn't a small sliver of the population still drive up the average price of whatever commodity the stimulus chooses to bring up, whether it be cars or cheese?

Admin Note: https://7sage.com/lsat_explanations/lsat-26-section-3-question-24/

Comments

  • Jordan JohnsonJordan Johnson Member
    edited July 2021 686 karma

    @Ashley2018 It seems like you recognized the Concept Shift in the stimulus, but are just having trouble applying it to the answer.

    In the stimulus, we're told:
    Avg price paid for a new car has increased (compared to avg individual income)

    But then the conclusion conflates price paid with individuals purchasing cars.

    It very well could be that the price paid has only increased for non-individuals (e.g. corporations), and thus individuals' price paid has remained the same relative to their individual income.

    Remember, it's a weakener, so it doesn't necessarily have to prove the conclusion 100% false.

    Hope that helps!

  • Ashley2018-1Ashley2018-1 Alum Member
    edited July 2021 2249 karma

    @"Jordan Johnson" said:
    @Ashley2018 It seems like you recognized the Concept Shift in the stimulus, but are just having trouble applying it to the answer.

    In the stimulus, we're told:
    Avg price paid for a new car has increased (compared to avg individual income)

    But then the conclusion conflates price paid with individuals purchasing cars.

    It very well could be that the price paid has only increased for non-individuals (e.g. corporations), and thus individuals' price paid has remained the same relative to their individual income.

    Remember, it's a weakener, so it doesn't necessarily have to prove the conclusion 100% false.

    Hope that helps!

    If the scenario in my original question were true, would that make the answer choice less of a weakener? If it’s just the amount that individuals or non individuals pay for the cars that drive up the price then why does the proportion of each group matter? Is it because the stimulus is talking about average price and not just exact price?

  • nnnnnnzzzznnnnnnzzzz Member
    177 karma

    @Ashley2018 said:

    @"Jordan Johnson" said:
    @Ashley2018 It seems like you recognized the Concept Shift in the stimulus, but are just having trouble applying it to the answer.

    In the stimulus, we're told:
    Avg price paid for a new car has increased (compared to avg individual income)

    But then the conclusion conflates price paid with individuals purchasing cars.

    It very well could be that the price paid has only increased for non-individuals (e.g. corporations), and thus individuals' price paid has remained the same relative to their individual income.

    Remember, it's a weakener, so it doesn't necessarily have to prove the conclusion 100% false.

    Hope that helps!

    If the scenario in my original question were true, would that make the answer choice less of a weakener? If it’s just the amount that individuals or non individuals pay for the cars that drive up the price then why does the proportion of each group matter? Is it because the stimulus is talking about average price and not just exact price?

    The opposite of weakening is strengthening. If you reduce the proportion of the people too much, the answer will become irrelevant or a strengthening answer and thus is incorrect.

    50/50 could be irrelevant
    Changing the answer to "Sales to individuals make up most (or more than 50%) of new car sales" makes the answer a strengthening answer.

  • LSAT LizardLSAT Lizard Alum Member
    edited July 2021 331 karma

    The opposite of weakening is strengthening. If you reduce the proportion of the people too much, the answer will become irrelevant or a strengthening answer and thus is incorrect.

    50/50 could be irrelevant
    Changing the answer to "Sales to individuals make up most (or more than 50%) of new car sales" makes the answer a strengthening answer.

    I disagree with both conclusions here. JY discusses the importance of the specific proportion of individual buyers to non-individual buyers in the part of the explanation video that discusses AC E (starts 2:40).

    The important takeaway is: as long as the proportion exists, the answer is correct. In the video JY changes AC E so it says: "Sales to individuals make up only a proportion of all new-car sales," then states that this AC would still be correct.

    Each of these hypothetical AC E's would be correct:

    • "Sales to individuals make up 99% of all car sales."
    • "Sales to individuals make up 50% of all car sales."
    • "Sales to individuals make up 1% of all car sales."
    • "Some car sales are made to buyers who are not individuals."

    In order for the argument in the stimulus to work, the author needs the assumption "100% of car buyers are individuals." So absolutely any AC challenging that assumption will weaken.


    In terms of one hypothetical answer being more weakening than the other- that kind of comparison gets subjective and messy very quickly. People have different opinions on what sorts of metrics determine one thing to be more or less weakening, and some people would dispute that two weakening things can differ in degree in the first place.

    My own approach to the topic is 'don't worry about it'. To get the question right, we don't care a single bit about whether one hypothetical answer is more weakening than another; the LSAT will never make us choose the most weakening AC out of two weakening ACs. This is a consequence of the one-right-answer principle.

  • nnnnnnzzzznnnnnnzzzz Member
    edited July 2021 177 karma

    @Tennysoj said:

    The opposite of weakening is strengthening. If you reduce the proportion of the people too much, the answer will become irrelevant or a strengthening answer and thus is incorrect.

    50/50 could be irrelevant
    Changing the answer to "Sales to individuals make up most (or more than 50%) of new car sales" makes the answer a strengthening answer.

    I disagree with both conclusions here. JY discusses the importance of the specific proportion of individual buyers to non-individual buyers in the part of the explanation video that discusses AC E (starts 2:40).

    The important takeaway is: as long as the proportion exists, the answer is correct. In the video JY changes AC E so it says: "Sales to individuals make up only a proportion of all new-car sales," then states that this AC would still be correct.

    Each of these hypothetical AC E's would be correct:

    • "Sales to individuals make up 99% of all car sales."
    • "Sales to individuals make up 50% of all car sales."
    • "Sales to individuals make up 1% of all car sales."
    • "Some car sales are made to buyers who are not individuals."

    In order for the argument in the stimulus to work, the author needs the assumption "100% of car buyers are individuals." So absolutely any AC challenging that assumption will weaken.


    In terms of one hypothetical answer being more weakening than the other- that kind of comparison gets subjective and messy very quickly. People have different opinions on what sorts of metrics determine one thing to be more or less weakening, and some people would dispute that two weakening things can differ in degree in the first place.

    My own approach to the topic is 'don't worry about it'. To get the question right, we don't care a single bit about whether one hypothetical answer is more weakening than another; the LSAT will never make us choose the most weakening AC out of two weakening ACs. This is a consequence of the one-right-answer principle.

    You are right. The answer choice compared the individuals with themselves of 25 years ago and through that we know the percentage of sales to individuals are shrinking, which means other types are increasing in term of percentages.

    As a result, changing the strength of the answer choice will still be correct.

  • Ashley2018-1Ashley2018-1 Alum Member
    edited July 2021 2249 karma

    @Tennysoj said:

    The opposite of weakening is strengthening. If you reduce the proportion of the people too much, the answer will become irrelevant or a strengthening answer and thus is incorrect.

    50/50 could be irrelevant
    Changing the answer to "Sales to individuals make up most (or more than 50%) of new car sales" makes the answer a strengthening answer.

    I disagree with both conclusions here. JY discusses the importance of the specific proportion of individual buyers to non-individual buyers in the part of the explanation video that discusses AC E (starts 2:40).

    The important takeaway is: as long as the proportion exists, the answer is correct. In the video JY changes AC E so it says: "Sales to individuals make up only a proportion of all new-car sales," then states that this AC would still be correct.

    Each of these hypothetical AC E's would be correct:

    • "Sales to individuals make up 99% of all car sales."
    • "Sales to individuals make up 50% of all car sales."
    • "Sales to individuals make up 1% of all car sales."
    • "Some car sales are made to buyers who are not individuals."

    In order for the argument in the stimulus to work, the author needs the assumption "100% of car buyers are individuals." So absolutely any AC challenging that assumption will weaken.


    In terms of one hypothetical answer being more weakening than the other- that kind of comparison gets subjective and messy very quickly. People have different opinions on what sorts of metrics determine one thing to be more or less weakening, and some people would dispute that two weakening things can differ in degree in the first place.

    My own approach to the topic is 'don't worry about it'. To get the question right, we don't care a single bit about whether one hypothetical answer is more weakening than another; the LSAT will never make us choose the most weakening AC out of two weakening ACs. This is a consequence of the one-right-answer principle.

    Yes, that was the basis of my original question. But doesn't it make it less likely that individuals were able to drive up the average price of cars if they only consisted of 1% of the ca-buying population than if they were 50% or 99% of that population? Are you saying this does this not matter because we only need to raise doubt and not prove that the conclusion is wrong? Like so long as the answer choice indicated that there was a group other than individuals purchasing cars, then the argument is weakened? Does that mean the argument was assuming that individuals are the only ones buying cars?

  • FindingSageFindingSage Alum Member
    2042 karma

    The original stimulus is making an assumption which is that individuals have been in the past and are currently the primary purchasers of cars. Answer choice E is calling out this assumption by pointing out that actually sales of cars to individuals make up a smaller proportion of sales to sales to individuals than they did 25 years ago. Does this mean that individuals are not the primary purchasers of cars anymore or that corporations or families are buying more cars than individuals? We really don't know and we don't need to. Answer Answer choice E just points out that perhaps the fact that the cars are higher priced relative to the increase in overall salaries isn't as big of a deal as the stimulus thinks it is because some of these cars for sale aren't intended for the individual to purchase anyway.
    On the surface this might look a portion/percentage question which often confuses people but it actually also features another cookie cutter flaw which we see in many questions, which is assuming what was true in the past will be true in the future. This is nicely pointed with answer choice E as part of the assumption that the argument is making. Maybe 25 years ago sales to individuals made up 75% of sales but what if they only make up 40% now? The LSAT tests this flaw alot- and does so using questions ranging from a one star question to a 5 star question like this one. This question is actually easier that it looks if you call out the flaw and have a strong prediction going into the answer choices and don't get tempted by some attractive trap answers.

  • Ashley2018-1Ashley2018-1 Alum Member
    edited July 2021 2249 karma

    @FindingSage said:
    The original stimulus is making an assumption which is that individuals have been in the past and are currently the primary purchasers of cars. Answer choice E is calling out this assumption by pointing out that actually sales of cars to individuals make up a smaller proportion of sales to sales to individuals than they did 25 years ago. Does this mean that individuals are not the primary purchasers of cars anymore or that corporations or families are buying more cars than individuals? We really don't know and we don't need to. Answer Answer choice E just points out that perhaps the fact that the cars are higher priced relative to the increase in overall salaries isn't as big of a deal as the stimulus thinks it is because some of these cars for sale aren't intended for the individual to purchase anyway.
    On the surface this might look a portion/percentage question which often confuses people but it actually also features another cookie cutter flaw which we see in many questions, which is assuming what was true in the past will be true in the future. This is nicely pointed with answer choice E as part of the assumption that the argument is making. Maybe 25 years ago sales to individuals made up 75% of sales but what if they only make up 40% now? The LSAT tests this flaw alot- and does so using questions ranging from a one star question to a 5 star question like this one. This question is actually easier that it looks if you call out the flaw and have a strong prediction going into the answer choices and don't get tempted by some attractive trap answers.

    Thank you for your input! So the argument is assuming that individuals are the primary purchasers of cars. But..does that mean that the specific proportions of answer choice E don't matter?

    PS. I have another LR and RC question down further in the feed...if you could take a look, I would be forever grateful!

  • FindingSageFindingSage Alum Member
    2042 karma

    The stimulus could actually be assuming either one and answer choice E would still be correct. If the stimulus was assuming that they were the ONLY purchasers, answer choice E would be pointing out actually they aren't. There are other people purchasing the cars as well ( families, corporations, small businesses...). If the stimulus was assuming that individuals are the primary purchasers than answer choice E would be pointing out that the proportion of individuals purchasing cars has decreased over the last 25 years.
    I took this test as a PT last week. Before I go into the answer choices I make a prediction/objection. I was thinking as a I read this argument: What if things have changed in the past 25 years? And I was also thinking, what if individuals aren't buying all of the cars? These objections lead me solidly to answer choice E in under target time. I did stop on answer choice A because at first glance it does appear to be making these same objections but reading it again, it actually doesn't do anything for the argument. Just because there has been a significant increase in households with more than one wage earner doesn't mean that these households are buying cars.

  • Ashley2018-1Ashley2018-1 Alum Member
    edited July 2021 2249 karma

    @FindingSage said:
    The stimulus could actually be assuming either one and answer choice E would still be correct. If the stimulus was assuming that they were the ONLY purchasers, answer choice E would be pointing out actually they aren't. There are other people purchasing the cars as well ( families, corporations, small businesses...). If the stimulus was assuming that individuals are the primary purchasers than answer choice E would be pointing out that the proportion of individuals purchasing cars has decreased over the last 25 years.
    I took this test as a PT last week. Before I go into the answer choices I make a prediction/objection. I was thinking as a I read this argument: What if things have changed in the past 25 years? And I was also thinking, what if individuals aren't buying all of the cars? These objections lead me solidly to answer choice E in under target time. I did stop on answer choice A because at first glance it does appear to be making these same objections but reading it again, it actually doesn't do anything for the argument. Just because there has been a significant increase in households with more than one wage earner doesn't mean that these households are buying cars.

    A was my original answer and I wanted it to fit so I tried stating that these households were buying cars and that they were splitting the bill so that individuals didn't have to fork out as much of their income as their counterparts did 25 years ago...is this stacking on too many assumptions? How do you draw the line?

  • Jordan JohnsonJordan Johnson Member
    686 karma

    @Ashley2018 said:
    A was my original answer and I wanted it to fit so I tried stating that these households were buying cars and that they were splitting the bill so that individuals didn't have to fork out as much of their income as their counterparts did 25 years ago...is this stacking on too many assumptions? How do you draw the line?

    In general, go with the answer choice that requires the least amount of inferences/assumptions. It sounds like you stacked two on top of each other to make answer choice A seem plausible to you, whereas E doesn't require the same mental gymnastics.

    Hope that helps!

Sign In or Register to comment.